Prove that $T$ is a linear operator












0












$begingroup$


Let $(v_n)_{ngeqslant1}$$l^2$ be a fixed sequence of real numbers.
Define a mapping T on $X=l^infty$ using the formula
$T(a_1,a_2,...) = (v_1a_1,v_2a_2,...),$ $x=(a_1,a_2,...)∈ l^infty$.



I have already proved that $Tx ∈ l^2$ for every $x∈l^infty$ for the first part of this question, the next part asks me to prove that
$$T:(l^infty, ||⋅||_infty)rightarrow (l^2,||⋅||_2)$$
is a linear operator.










share|cite|improve this question











$endgroup$












  • $begingroup$
    Is your question on the next part? What is your question? Also if that is your question, what have you tried?
    $endgroup$
    – jgon
    Dec 9 '18 at 21:29










  • $begingroup$
    Sorry if it wasn't clear. My question is the next part, verifying that T is a linear operator, the notes I have aren't clear but I've tried to do this using the definition but it's not really working out.
    $endgroup$
    – callista
    Dec 9 '18 at 21:46
















0












$begingroup$


Let $(v_n)_{ngeqslant1}$$l^2$ be a fixed sequence of real numbers.
Define a mapping T on $X=l^infty$ using the formula
$T(a_1,a_2,...) = (v_1a_1,v_2a_2,...),$ $x=(a_1,a_2,...)∈ l^infty$.



I have already proved that $Tx ∈ l^2$ for every $x∈l^infty$ for the first part of this question, the next part asks me to prove that
$$T:(l^infty, ||⋅||_infty)rightarrow (l^2,||⋅||_2)$$
is a linear operator.










share|cite|improve this question











$endgroup$












  • $begingroup$
    Is your question on the next part? What is your question? Also if that is your question, what have you tried?
    $endgroup$
    – jgon
    Dec 9 '18 at 21:29










  • $begingroup$
    Sorry if it wasn't clear. My question is the next part, verifying that T is a linear operator, the notes I have aren't clear but I've tried to do this using the definition but it's not really working out.
    $endgroup$
    – callista
    Dec 9 '18 at 21:46














0












0








0





$begingroup$


Let $(v_n)_{ngeqslant1}$$l^2$ be a fixed sequence of real numbers.
Define a mapping T on $X=l^infty$ using the formula
$T(a_1,a_2,...) = (v_1a_1,v_2a_2,...),$ $x=(a_1,a_2,...)∈ l^infty$.



I have already proved that $Tx ∈ l^2$ for every $x∈l^infty$ for the first part of this question, the next part asks me to prove that
$$T:(l^infty, ||⋅||_infty)rightarrow (l^2,||⋅||_2)$$
is a linear operator.










share|cite|improve this question











$endgroup$




Let $(v_n)_{ngeqslant1}$$l^2$ be a fixed sequence of real numbers.
Define a mapping T on $X=l^infty$ using the formula
$T(a_1,a_2,...) = (v_1a_1,v_2a_2,...),$ $x=(a_1,a_2,...)∈ l^infty$.



I have already proved that $Tx ∈ l^2$ for every $x∈l^infty$ for the first part of this question, the next part asks me to prove that
$$T:(l^infty, ||⋅||_infty)rightarrow (l^2,||⋅||_2)$$
is a linear operator.







real-analysis functional-analysis






share|cite|improve this question















share|cite|improve this question













share|cite|improve this question




share|cite|improve this question








edited Dec 10 '18 at 3:50









user1101010

7171730




7171730










asked Dec 9 '18 at 21:20









callistacallista

32




32












  • $begingroup$
    Is your question on the next part? What is your question? Also if that is your question, what have you tried?
    $endgroup$
    – jgon
    Dec 9 '18 at 21:29










  • $begingroup$
    Sorry if it wasn't clear. My question is the next part, verifying that T is a linear operator, the notes I have aren't clear but I've tried to do this using the definition but it's not really working out.
    $endgroup$
    – callista
    Dec 9 '18 at 21:46


















  • $begingroup$
    Is your question on the next part? What is your question? Also if that is your question, what have you tried?
    $endgroup$
    – jgon
    Dec 9 '18 at 21:29










  • $begingroup$
    Sorry if it wasn't clear. My question is the next part, verifying that T is a linear operator, the notes I have aren't clear but I've tried to do this using the definition but it's not really working out.
    $endgroup$
    – callista
    Dec 9 '18 at 21:46
















$begingroup$
Is your question on the next part? What is your question? Also if that is your question, what have you tried?
$endgroup$
– jgon
Dec 9 '18 at 21:29




$begingroup$
Is your question on the next part? What is your question? Also if that is your question, what have you tried?
$endgroup$
– jgon
Dec 9 '18 at 21:29












$begingroup$
Sorry if it wasn't clear. My question is the next part, verifying that T is a linear operator, the notes I have aren't clear but I've tried to do this using the definition but it's not really working out.
$endgroup$
– callista
Dec 9 '18 at 21:46




$begingroup$
Sorry if it wasn't clear. My question is the next part, verifying that T is a linear operator, the notes I have aren't clear but I've tried to do this using the definition but it's not really working out.
$endgroup$
– callista
Dec 9 '18 at 21:46










1 Answer
1






active

oldest

votes


















1












$begingroup$

Hint:



The operator $T$ is linear if the following hold for $s,tin l^infty$, $cinmathbb{R}$:



1) $quad$ $T(s+t) = T(s) + T(t)$



2) $quad$ $T(ccdot s) = ccdot T(s)$



So



$T(s+t) = (v_1(s_1+t_1),v_2(s_1+t_1),...)$ by definition of the operator, can you gontinue from here?






share|cite|improve this answer









$endgroup$













  • $begingroup$
    Oh right, so do I only really need to show that the conditions of linearity are met?
    $endgroup$
    – callista
    Dec 9 '18 at 21:50










  • $begingroup$
    Yes, that is right.
    $endgroup$
    – Bo5man
    Dec 10 '18 at 8:48











Your Answer





StackExchange.ifUsing("editor", function () {
return StackExchange.using("mathjaxEditing", function () {
StackExchange.MarkdownEditor.creationCallbacks.add(function (editor, postfix) {
StackExchange.mathjaxEditing.prepareWmdForMathJax(editor, postfix, [["$", "$"], ["\\(","\\)"]]);
});
});
}, "mathjax-editing");

StackExchange.ready(function() {
var channelOptions = {
tags: "".split(" "),
id: "69"
};
initTagRenderer("".split(" "), "".split(" "), channelOptions);

StackExchange.using("externalEditor", function() {
// Have to fire editor after snippets, if snippets enabled
if (StackExchange.settings.snippets.snippetsEnabled) {
StackExchange.using("snippets", function() {
createEditor();
});
}
else {
createEditor();
}
});

function createEditor() {
StackExchange.prepareEditor({
heartbeatType: 'answer',
autoActivateHeartbeat: false,
convertImagesToLinks: true,
noModals: true,
showLowRepImageUploadWarning: true,
reputationToPostImages: 10,
bindNavPrevention: true,
postfix: "",
imageUploader: {
brandingHtml: "Powered by u003ca class="icon-imgur-white" href="https://imgur.com/"u003eu003c/au003e",
contentPolicyHtml: "User contributions licensed under u003ca href="https://creativecommons.org/licenses/by-sa/3.0/"u003ecc by-sa 3.0 with attribution requiredu003c/au003e u003ca href="https://stackoverflow.com/legal/content-policy"u003e(content policy)u003c/au003e",
allowUrls: true
},
noCode: true, onDemand: true,
discardSelector: ".discard-answer"
,immediatelyShowMarkdownHelp:true
});


}
});














draft saved

draft discarded


















StackExchange.ready(
function () {
StackExchange.openid.initPostLogin('.new-post-login', 'https%3a%2f%2fmath.stackexchange.com%2fquestions%2f3033029%2fprove-that-t-is-a-linear-operator%23new-answer', 'question_page');
}
);

Post as a guest















Required, but never shown

























1 Answer
1






active

oldest

votes








1 Answer
1






active

oldest

votes









active

oldest

votes






active

oldest

votes









1












$begingroup$

Hint:



The operator $T$ is linear if the following hold for $s,tin l^infty$, $cinmathbb{R}$:



1) $quad$ $T(s+t) = T(s) + T(t)$



2) $quad$ $T(ccdot s) = ccdot T(s)$



So



$T(s+t) = (v_1(s_1+t_1),v_2(s_1+t_1),...)$ by definition of the operator, can you gontinue from here?






share|cite|improve this answer









$endgroup$













  • $begingroup$
    Oh right, so do I only really need to show that the conditions of linearity are met?
    $endgroup$
    – callista
    Dec 9 '18 at 21:50










  • $begingroup$
    Yes, that is right.
    $endgroup$
    – Bo5man
    Dec 10 '18 at 8:48
















1












$begingroup$

Hint:



The operator $T$ is linear if the following hold for $s,tin l^infty$, $cinmathbb{R}$:



1) $quad$ $T(s+t) = T(s) + T(t)$



2) $quad$ $T(ccdot s) = ccdot T(s)$



So



$T(s+t) = (v_1(s_1+t_1),v_2(s_1+t_1),...)$ by definition of the operator, can you gontinue from here?






share|cite|improve this answer









$endgroup$













  • $begingroup$
    Oh right, so do I only really need to show that the conditions of linearity are met?
    $endgroup$
    – callista
    Dec 9 '18 at 21:50










  • $begingroup$
    Yes, that is right.
    $endgroup$
    – Bo5man
    Dec 10 '18 at 8:48














1












1








1





$begingroup$

Hint:



The operator $T$ is linear if the following hold for $s,tin l^infty$, $cinmathbb{R}$:



1) $quad$ $T(s+t) = T(s) + T(t)$



2) $quad$ $T(ccdot s) = ccdot T(s)$



So



$T(s+t) = (v_1(s_1+t_1),v_2(s_1+t_1),...)$ by definition of the operator, can you gontinue from here?






share|cite|improve this answer









$endgroup$



Hint:



The operator $T$ is linear if the following hold for $s,tin l^infty$, $cinmathbb{R}$:



1) $quad$ $T(s+t) = T(s) + T(t)$



2) $quad$ $T(ccdot s) = ccdot T(s)$



So



$T(s+t) = (v_1(s_1+t_1),v_2(s_1+t_1),...)$ by definition of the operator, can you gontinue from here?







share|cite|improve this answer












share|cite|improve this answer



share|cite|improve this answer










answered Dec 9 '18 at 21:36









Bo5manBo5man

517




517












  • $begingroup$
    Oh right, so do I only really need to show that the conditions of linearity are met?
    $endgroup$
    – callista
    Dec 9 '18 at 21:50










  • $begingroup$
    Yes, that is right.
    $endgroup$
    – Bo5man
    Dec 10 '18 at 8:48


















  • $begingroup$
    Oh right, so do I only really need to show that the conditions of linearity are met?
    $endgroup$
    – callista
    Dec 9 '18 at 21:50










  • $begingroup$
    Yes, that is right.
    $endgroup$
    – Bo5man
    Dec 10 '18 at 8:48
















$begingroup$
Oh right, so do I only really need to show that the conditions of linearity are met?
$endgroup$
– callista
Dec 9 '18 at 21:50




$begingroup$
Oh right, so do I only really need to show that the conditions of linearity are met?
$endgroup$
– callista
Dec 9 '18 at 21:50












$begingroup$
Yes, that is right.
$endgroup$
– Bo5man
Dec 10 '18 at 8:48




$begingroup$
Yes, that is right.
$endgroup$
– Bo5man
Dec 10 '18 at 8:48


















draft saved

draft discarded




















































Thanks for contributing an answer to Mathematics Stack Exchange!


  • Please be sure to answer the question. Provide details and share your research!

But avoid



  • Asking for help, clarification, or responding to other answers.

  • Making statements based on opinion; back them up with references or personal experience.


Use MathJax to format equations. MathJax reference.


To learn more, see our tips on writing great answers.




draft saved


draft discarded














StackExchange.ready(
function () {
StackExchange.openid.initPostLogin('.new-post-login', 'https%3a%2f%2fmath.stackexchange.com%2fquestions%2f3033029%2fprove-that-t-is-a-linear-operator%23new-answer', 'question_page');
}
);

Post as a guest















Required, but never shown





















































Required, but never shown














Required, but never shown












Required, but never shown







Required, but never shown

































Required, but never shown














Required, but never shown












Required, but never shown







Required, but never shown







Popular posts from this blog

How do I know what Microsoft account the skydrive app is syncing to?

Grease: Live!

When does type information flow backwards in C++?